2
$\begingroup$

I have spent more time thinking than I would like to admit because of the following sentence: "Choose $t = c_6X^{1/e}$ and we obtain Theorem 2.". What obvious argument am I missing that allows us to obtain Theorem 2?

The sentence, Theorem 2, and supporting background come from the first four pages of a paper of P. Erdős and J. Selfridge, found at https://www.math-inst.hu/~p_erdos/1971-24.pdf (Some problems on the prime factors of consecutive integers II). I will interpret some of it here, hopefully enough to explain the situation.

"permutation coefficients" in the question title refers to numbers of the form $W=W(n,k)=\prod_{1\leq i \leq k}(n+i)$, and the paper deals with the number $v(n,k)$ of distinct prime factors of $W$, and related quantities. Let $f_0(n)$ be the largest positive integer $k$ such that $v(n,k) \geq k$, and let $f_1(n)$ be the smallest $k$ such that for $1 \leq j \leq k$ we have $v(n,j) \geq j$, and also $v(n,k+1)=k$. It takes a little effort to show $f_0(n) \geq f_1(n)$, and more to show strict inequality occurs for an infinite number of $n$, with the smallest occurring above $n=5000$. $f_1$ is provided for completeness; it will be enough for answering this question to restrict attention to $f_0(n)$.

After some observations and a nice (but also telegraphic) proof of an upper bound for $f_0(n)$, we come to theorem 2, which states: for infinitely many $n$, $$ f_0(n) \lt c_6n^{1/e} \textrm{ and } f_1(n) \lt c_7n^{1/e}. $$ Here the symbols $c_6$ and $c_7$ refer to absolute constants independent of $n$ or anything else except the mind of the proof composer/interpreter. I interpret some of the proof below.

It is enough to consider the proposition for $n$ sufficiently large, so choose $X$ big, and consider for a sufficiently small constant $c_8$ the set $L$ of prime numbers in the interval $(c_8X^{1/e}, X)$. Where the authors write $u(m,X)$ I will write $u(m)$, which counts the number of (distinct) prime factors $m$ has that come from $L$. Then it is noted that in total, there are more instances of these prime factors among $m \in [1,X]$ than the numbers in that interval. To wit: $$ \sum_{1 \leq m \leq X} u(m) = \sum_{p \in L} \lfloor X/p \rfloor \gt X \sum_{ p \in L } 1/p - \pi(X) \gt X. $$ From this they observe that there is an integer $m \in (c_8X^{1/e}, X - c_8X^{1/e})$ such that the following inequality holds for all $t$ with $1\leq t \leq X -m$: $$\sum_{1\leq i \leq t} u(m+i) \geq t.$$

"Choose $t = c_6X^{1/e}$ and we obtain Theorem 2.". Really? How so?

I think what is provided is too abbreviated, and in an answer below I provide what I think is a reasonable expansion. However, it seems too long, and I wonder if there is something simpler and more direct. There is a little more that follows but it does not especially enlighten me. In particular I can't determine which of $c_6$ and $c_8$ is larger in general, although I suspect $c_6$ is smaller.

Gerhard "Not A Student Of Erdős" Paseman, 2017.09.07.

$\endgroup$
2
  • $\begingroup$ The promised answer will come tomorrow. In brief, I think X should be changed and c_8 carefully chosen so that the inequality sum u(m) greater than or equal to t holds for t from 1 to X-m, but fails at t =X-m+1. Also, it makes sense to choose c_6 greater than c_8. Gerhard "May Think Differently Tomorrow Morning" Paseman, 2017.09.07. $\endgroup$ Sep 8, 2017 at 4:20
  • $\begingroup$ It just occurred to me to try n=m+t. If this works, it may turn out to be the desired expansion. Gerhard "If So, Apologies To Authors" Paseman, 2017.09.08. $\endgroup$ Sep 8, 2017 at 21:00

2 Answers 2

2
$\begingroup$

The simplest explanation is that it is a mistake. One can however complete the proof as follows:

If $X$ is large enough then $u(m) \in \{ 0,1,2 \}$ for all $m$ since $e < 3$. Thus the number of distinct primes in $L$ dividing $m$ is at least $u(m)$, unless $u(m) = 2$ and $m$ has only one prime factor in $L$, in which case $m = p^2 q$.

Let us count pairs of integers $(p,q)$ where $p > R := c_{8} X^{\frac{1}{e}}$ and $m < p^2 q \leq m+t$. This it at most $$ \sum_{q \leq X R^{-2}} \mathrm{number \ of \ squares \ in \ } ]mq^{-1},(m+t)q^{-1} ] \\ \leq \sum_{q \leq X R^{-2}} \left( \frac{t}{\sqrt{qm}} + O(1) \right) \\ \ll \frac{t}{\sqrt{R}} \sqrt{X R^{-2}} + X R^{-2} \\ \ll t X^{-\alpha} +X^{\frac{1}{e} - 2 \alpha} $$ where $\alpha = \frac{3-e}{2e} >0$.

Thus the number of distinct prime factors of $\prod_{j=1}^{t} (m+j)$ in $L$ is at least $t - O(t X^{-\alpha} +X^{\frac{1}{e} - 2 \alpha})$. By choosing $t$ of size $X^{\frac{1}{e}}$, one thus gets $\geq t - O(t X^{-\alpha})$ distinct prime factors $>t$. By adding the $\pi(t)$ primes $\leq t$, we get $\geq t$ distinct prime factors (for $X$ large enough).

EDIT: As pointed out Gerhard Paseman below, I answered a different question ... The original question can be answered as follows :

Let $\omega_{>k}(n)$ be the number of distinct prime factors $>k$ of $n$. We first note that $$ \nu(n,k) = \pi(k) + \sum_{i=1}^k \omega_{>k}(n+i). $$ Let $c >0$ be large enough so that $$ \sum_{X<n \leq 2X + X^{\frac{4}{5}}} \omega_{>R}(n) \leq \left( 1 - \frac{3e}{\log X} \right) X $$ holds for large $X$ with $R = c X^{\frac{1}{e}}$ (indeed the inequality holds with $3e$ replaced by $e \log c + O(1)$). We first show that for any $X$ large enough the following holds:

$(*)$ there exists a $n \in [X,2X]$ such that for each $k \in [R, X^{\frac{4}{5}}]$, one has $\nu(n,k) < k$.

Indeed, assume the contrary for some $X$. Then starting with $n_0 =X$, we get a $k_0 \in [R, X^{\frac{4}{5}}]$ such that $\nu(n_0,k_0) \geq k_0$, and then with $n_1 = n_0 + k_0$ some $k_1$ such that $\nu(n_1,k_1) \geq k_1$, .... and so on until $n_{J+1} = n_J + k_J > 2X$ for some $J$. We then have $$ \sum_{n_j < n \leq n_{j+1}} \omega_{>R}(n) \geq \sum_{n_j < n \leq n_{j+1}} \omega_{>k_j}(n) \geq k_j - \pi(k_j) \geq \left( 1 - \frac{2}{\log R} \right) k_j. $$ Summing over $j$, this yields $$ \sum_{X<n \leq 2X + X^{\frac{4}{5}}} \omega_{>R}(n) \geq \left( 1 - \frac{2}{\log R} \right) X,$$ which contradicts our choice of $c$ for $X$ large enough.

Thus for any $X$ large enough one can take $n \in [X,2X]$ as in $(*)$. One has $\nu(n,k) < k$ for $k \in [R, X^{\frac{4}{5}}]$. But for $k > X^{\frac{4}{5}}$, a direct count using Brun-Titschmarsh inequality yields $\nu(n,k) \leq c k + o(k)$ with $c = 2 \log \frac{4}{3} < 1$, hence $\nu(n,k) < k$ when $X$ is large enough. Thus $f_0(n) < R \leq c n^{\frac{1}{e}}$.

$\endgroup$
6
  • $\begingroup$ Yes, but the theorem says there is n with W(n,t) having fewer prime factors (than large t). Does this argument find such an n? (Even though it is challenging, I believe I can show the existence of $m$ claimed in the proof.) Gerhard "One Fewer Hump Is Needed" Paseman, 2017.09.07. $\endgroup$ Sep 8, 2017 at 14:24
  • $\begingroup$ What do you mean by "fewer prime factors than large $t$" ? $\endgroup$
    – js21
    Sep 11, 2017 at 8:07
  • $\begingroup$ I mean that you are showing W(m,t) has at least t distinct prime factors, which I believe is true for small t, but the result says for large t that W(n,t) (maybe even W(m,t)) has fewer than t distinct prime factors. Or have I gotten things turned around? Gerhard "Maybe Flatten One Extra Hump?" Paseman, 2017.09.11. $\endgroup$ Sep 11, 2017 at 11:11
  • $\begingroup$ Oh, you are absolutely right! I was confused by the inequalities in the paper; they seem to go the wrong way now. I just added to my answer a proof of the theorem obtained by reversing inequalities... $\endgroup$
    – js21
    Sep 11, 2017 at 14:40
  • $\begingroup$ Thanks for the edit; I will study it carefully. Does one need Brun-Titchmarsh to carry this through? (I am hoping for more combinatorics and less analytics for a fill in.) Also, >k and >k, or do you really mean >k and >R in your defn of omega_>k? Gerhard "We All Have Our Preferences" Paseman, 2017.09.11. $\endgroup$ Sep 11, 2017 at 14:51
0
$\begingroup$

I think the combinatorial argument could be more useful if made more clear. I want to make sure I am using it correctly; consider this answer an extension of the question and asking for other uses of this argument in the literature.

What I call $u(m)$ (and the authors call $u(m,X)$) is a nice counting function. For sufficiently large positive integers $m \leq X$, $u(m)$ takes on the values $0,1,$ and $2$. One chooses the exponent $1/e$ to get the summatory function $\sum_{1\leq m \leq X} u(m)$ up to a value close to and above $X$, and indeed $c_8$ does not need to be much smaller than $1/e$ to accomplish this, so one can make the argument with an explicit choice. One can also define $L$ so that the sum is within a small constant times $X^{1/e}$ from $X$. (We can also vary $X$; this will be crucial later.)

For arbitrary $ X$, one can show the existence of the stated $m$ as follows. Consider for any positive integer $j$ less than $X$ the largest $s$ such that $\sum_{1 \leq i \leq r} u(j+i) \geq r$ holds for $1 \leq r \leq s$ but does not hold for $r=s+1$. Indeed, for small $j$ outside of $L$ $u(j+i)$ is $0$ for small $i$, and so for these $j$ $s$ is $0$. One can partition $[1,X]$ into disjoint intervals of the form $[j,j+s]$, and there will be at least $c_8X^{1/e}$ many of these intervals, which means (since the summatory exceeds $X$) that the excess $u$ values pile up later on in the interval. $m$ is the largest of these $j$'s and has to handle all the excess "missed out" by the smaller $j$'s, so we must have $m \in (c_8X^{1/e}, X - c_8X^{1/e})$. This is argument has less control if you replace $1/e$ by something smaller, say $1/10$. You can show the existence of $m$, but $u$ would become more bumpy and you might not prove as large a value of $s$ as here ($m$ might end up too close to $X$.)

As currently written, it is not clear if the authors expect $\sum_{1 \leq i \leq X+1-m} u(m+i) \geq X +1 -m$ to fail. Indeed, if it does not fail, then $v(m,X-m+1)$ may be larger than $ X-m+1 $ just because $W(m,X-m+1)$ might have that many prime factors bigger than $c_8X^{1/e}$ alone, and still more small prime factors. Then $f_0(m)$ would be larger than $X-m+1$, and $m$ might not be one of the infinitely many $n$ highlighted in Theorem 2. More importantly, the suggested value of $n$ of $X-t$ or $n= X - c_6X^{1/e}$ might also not satisfy $f_0(n) \lt c_6X^{1/e}$ because we found a large interval full of primes and semiprimes containing $X$. This issue as well as having to fill in details like "where is $n$?" prompts the question.

If we can guarantee that the last inequality fails for $t= X-m+1$, then I can finish the proof easily. Choose $c_6$ and $t$ as suggested, and choose $n=X-t$. Then $W(n,t)$ has fewer than $t$ factors coming from $L$ by the construction of $m$ and the guarantee. If also $c_6$ is larger than $c_8$ and $n \gt m$, then some of the factors from $L$ are repeated (so not distinct) and since we can vary $t$ between $1$ and $X-m$, we can choose $c_6$ so that there are more repeats than primes below $c_8X^{1/e}$, and now we get finally an $n$ that gets close to the goal, having $v(n, X-n) $ less than $c_6X^{1/e}$. To get the goal requires a little more tweaking of $c_6$, but we have made assumptions on $X$ and $m$, so let's address those.

So let us not choose arbitrary $X$. Let us choose $X' \geq X$ so that (now renaming $X'$ to $X$) we can assert the existence of $m \lt X$ above where the inequality in $t$ above not only works for $1 \leq t \leq X-m$ and also fails at $t=X+1-m$. This is essentially the partition argument above, except we choose $X'$ to be $m+s$ (or $m+s+1$, whichever works) and rename it to $X$. We can also tweak $c_8$ and $L$ correspondingly to get this to work.

Here is the key point: for all $n$ in $(m, X)$ now we have $\sum_{1 \leq i \leq X-n} u(n+i) \lt X-n$, and now we have the inequality pointing in the right direction (because of the failure for $m$ at $t=X+1-m$). Now we can have our pick of $n$ to get the small primes in $W(n,X-n)$ outnumbered by the repeats in $L$. Even if $m$ is too small, we can start "backing up" (by choosing $m' \lt m$ and renaming) and find a good value for $n$ less than the original $m$. There are more details to consider, but now I see a light at the end of the tunnel, and it does not look like an oncoming train.

I can understand leaving detail to the reader, but I think the reader deserves the bone of the phrase ("and let $n=X-t$"). Unless, of course, I missed something simpler.

Gerhard "Maybe Can Hitch A Ride?" Paseman, 2017.09.08.

$\endgroup$
1
  • $\begingroup$ I suspect the above idea fails because X' will end up being much larger than X. It may not matter, as m may be about X^(2/e) away from X, in which case a different argument may work. Gerhard "Switching To Plan C Now" Paseman, 2017.09.09. $\endgroup$ Sep 9, 2017 at 22:58

Your Answer

By clicking “Post Your Answer”, you agree to our terms of service and acknowledge you have read our privacy policy.

Not the answer you're looking for? Browse other questions tagged or ask your own question.